ChaseDream
搜索
返回列表 发新帖
查看: 4399|回复: 7
打印 上一主题 下一主题

KAPLAN 逻辑题及答案 2009

[复制链接]
楼主
发表于 2009-8-31 21:57:00 | 只看该作者

KAPLAN 逻辑题及答案 2009

 本人扫描水平很欠,也没有时间修改,如果你不介意先看着,等我有时间在修改。

Kaplan的最新版的题,跟大家share 一下。

我个人认为kaplan 的逻辑部分很好。

沙发
发表于 2009-9-1 16:47:00 | 只看该作者

Grace同学,哪里有得看?

板凳
 楼主| 发表于 2009-9-9 00:34:00 | 只看该作者
谁知道如何正确上传?我为什么每次都是无法上传呢?是否是因为在国外的原因呢?或者逻辑版主请给我你的邮箱,我发给你,你再上传好了。谢谢
很抱歉,让众多人失望
地板
 楼主| 发表于 2009-9-9 00:36:00 | 只看该作者

Chapter 5: GMAT Critical Reasoning

We hope you're in a contentious mood, because people who love to argue are likely to do well

in Critical Reasoning, our last Verbal question type. This is because Critical Reasoning is all about

arguments, and your ability to break them apart, identify their weak spots, attack them, defend

them-do what you will  with them.

“why are arguments so darn important?" you may well ask. Good question: This means you're not

taking things lying down. This means you're getting in a mood to argue ! The answer is that the ability

to argue persuasively-to understand the logic of an argument, evaluate its merits, and be able to

respond to its strengths and weaknesses-requires the critical thinking skills that any good business

"manager should have. Business managers need to be able to evaluate arguments and proposals with

  critical eye, After all, not all business proposals are created equal!

The 41-question Verbal Section contains about 12 or 15 Critical Reasoning questions, so Critical

reasoning will account for about 30 percent of your Verbal score. We will show you how to evaluate

critical Reasoning arguments soon, but the first thing you need to do is familiarize yourself with this

question type.

ANATOMY OF A CRITICAL REASONING QUESTION(剖析CR问题)

Let's start by looking at the various parts of a typical Critical Reasoning question.

The Directions

the directions for this question type are short and sweet:

Directions: For each question, select the best of the answer choices given.

The directions are straightforward, but note once again that they ask you for the best answer-not the

effect answer.

The Stimulus

Above every question you'll find what we at Kaplan call the stimulus

1.   A recent study has concluded that, contrary to the claims of those trying to ban cigarette

     advertisements altogether, cigarette ads placed on billboards and in magazines have little

     to no effect on the smoking habits of the smokers who view the ads. The study, which

     surveyed more than 20,000 smokers and solicited their reasons for continuing to smoke,

     found that practically no one in the survey felt that these advertisements influenced their

     decision to smoke.

This is a short passage, typically in the form of an argument, which may be drawn from many possible

areas, including the natural sciences, the humanities, business, and even casual conversation. Even if

the subject matter seems strange, relax. All the information you need to answer the question is on the

screen. You don't need to bring in any outside knowledge.

The Question and Answer Choices

Here's the question:

The study's conclusion is based upon which of the following assumptions?

Read this first/Here's where you find out your task. You may need to identify an argument's

assumption or flaw; you may need to determine what would strengthen or weaken the argument; or

you may be asked to make a deduction. Pay close attention here: If you misinterpret the question, all

your other work will go to waste.

    A.  People do not switch cigarette brands based on their exposure to cigarette ads on

     billboards and in magazines.

B.  Cigarette ads on billboards and in magazines do not encourage nonsmokers to take

     up the habit.

C.  Banning cigarette ads altogether will encourage people to give up smoking.

D.  People are consciously aware of all the reasons they choose to smoke.

E.  People who decide to smoke do so for rational reasons.

And these are the answer choices. As you no doubt know by now, the test makers have set up one

(and only one) of these choices to be correct. If you understand the question and are able to follow

the argument, you should be able to zero in on the correct answer quickly. Wrong answer choices

usually distort the text, misrepresent its scope, or are the opposite of what you're looking for. You now know the basics of the Kaplan method for Critical Reasoning.

THE KAPLAN FOUR-STEP METHOD FOR CRITICAL REASONING

Here's our four-step method for handling this question type :

I. Read the question first.

You should always read the question before you look at the stimulus, because how you read and

attack the stimulus will vary depending upon the task. If you know what you're supposed to do to the

stimulus, you'll be able to focus your reading on the task ahead.

2. Read the stimulus.

Once you know your task, read the stimulus, paraphrasing as you go. Most, but not all, Critical

Reasoning questions will require you to identify the parts of an argument. The key is to read actively

and critically. Get a sense of how strong or weak the argument is.

3. Try to prephrase an answer,

Sometimes, if you've read the stimulus critically enough, you'll know the answer without even looking

at the answer choices. Other times, you'll just have a general idea of the answer may say. Either way,

you'll have an easier time finding the correct answer if you have a sense of what you're looking for.

4. Attack the answer choices

If you were able to prephrase an answer, skim the choices looking for a match. If you couldn't come

up with a prephrase, read and evaluate the answer choices. You can always eliminate answer choices

that stray beyond the scope of the stimulus. Other eliminating strategies will vary depending upon the

question asked. Once you've settled on an answer, you may want to reread the question quickly to

confirm that you've answered the question that was asked.

BREAKING DOWN AN ARGUMENT

As noted, the typical Critical Reasoning stimulus is in the form of an argument, and the typical Critical

Reasoning question will require you to understand the argument, which involves being able to identify

its component parts. But just what do we mean by "argument" on the GMAT? What we do not mean is

a disagreement involving loud and abusive language, sometimes requiring intervention by the police.

On the GMAT, an argument is an attempt at a reasoned appeal-it's a piece of text in which an author

puts forth a claim and tries to support it. Thus, every GMAT argument contains the following parts:

·         The conclusion (the author's claim; the point he or she is trying to make)

·          The evidence (the support the author offers for the conclusion)

Identifying the Conclusion and Evidence

For starters, you don't want to make the mistake of thinking that the conclusion must come at the end

of the stimulus. A conclusion could be the first sentence, followed by the evidence, or it could be the

last sentence, with the evidence preceding it. Since doing well on Critical Reasoning requires being   

able to identify an argument's component parts, we must have a way to distinguish the argument's

conclusion from its evidence. Fortunately, there are several.

Keywords

In chapter 3, we discussed the concept of Keywords, the structural clues that authors use to signal

logical connections between ideas. Luckily, there are distinct Conclusion Keywords and distinct

Evidence Keywords, and even luckier still, the GMAT writers use these Keywords frequently. Here are

the most important ones to watch out for:

Conclusion Keywords: Therefore  Thus  As a result  Hence  Clearly  So  Consequently

Evidence Keywords:   because   since  for

Note: If a sentence contains an Evidence Keyword, the part with the Evidence Keyword contains

evidence; the rest of the sentence contains the conclusion.

But what if the structure of the argument isn't handed to you on a platter? Sometimes, there are no

explicit Keyword signals in the argument. Fortunately, there are other ways to skin an argument.

The "One Sentence" Test

As already noted, an argument's conclusion is the author's main point. It's the belief the author wants

the reader to adopt, or the actions she wants him to take. Evidence, on the other hand, is just there

to support the conclusion; a piece of evidence tends to sound like an undisputed fact rather than a

strongly held view.

Therefore, if no obvious Keyword points the way to the conclusion, give the stimulus the One

Sentence Test: Ask yourself what statement the author would make if limited to a single sentence. The

statement you come up with (which may sound like a recommendation or an order) should be the

conclusion.

The "What" versus "Why" Test

There's one last technique that can help you to distinguish between evidence and conclusion. It's

based on the fact that the evidence and the conclusion answer two different types of questions, as you

can see

This part:   Answers the question:

Conclusion  What does the author believe (should be done)?

Evidence    Why does the author believe this?

So if you're unsure whether a particular statement is evidence or a conclusion, ask yourself: Does this

sentence express what the author believes? Or, does it explain why the author believes it?

Identify the conclusions in the following arguments:

1. Get Going magazine surveyed its readers and found that three out of every four people who want

  to visit the Andaman Islands wish to do so on a package tour provided by a tour operator. Since

  tour operators currently in business can provide tours for only 60 percent of those who wish to

  visit the Andaman Islands, setting up a travel company that offers tours to the Andaman Islands is

  a relatively risk-free way to make money.

2. In a recently published study, researchers concluded that steady chromium intake, even in small

   doses, can significantly lower the life expectancy of some primates. In the study, 250 chimpanzees

   were reared in a laboratory and given small but constant doses of chromium. Within 17 years, all

   of the chimps had died. Moreover, their average lifespan was just over 11 years, whereas the aver-

   age lifespan for untreated chimps in the wild is approximately 40 years.

Once you locate the conclusion, the rest of the stimulus should be evidence that supports the

conclusion. We'll see whether you've properly identified the conclusions in a bit. But there's a third

crucial component to every GMAT argument. Did you find the evidence in the above arguments

strong enough to convince you of the conclusions? Or, did you find that additional evidence would

be necessary for the argument to be convincing? This additional evidence-facts that need to be, but

haven't been, established or mentioned-has a special name.

Assumptions

There are very few vocabulary words you need to know for the GMAT, but here's one of them:

Assumptions act as additional, unstated evidence. They are statements that must be true for the

argument to be valid.

To find assumptions, compare the terms in the conclusion with the terms in the evidence. Then ask

yourself: "What's missing?" Almost every GMAT argument contains a logical gap between the evidence

and the conclusion. The author's assumptions are what fills that gap. With practice, as you read a

GMAT argument, the author's assumptions should start to jump right out at you. This is because the

same types of assumptions appear over and over again on GMAT arguments.

Let's see how well you do at spotting assumptions. Take another look at the two arguments you just

analyzed. Weigh the conclusions you identified against the evidence, and ask yourself what's missing

Remember to read critically. Put the author's assumptions in your own words and then see whether

you identified the same assumptions (and conclusions) that we did.

1. The Andaman Islands Argument.

The author assumes that  _____________________________________________.

2. The Chromium Chimps Argument.

The author assumes that ________________________________________________.

Answers and Explanations

1. The Andaman Islands Argument: We hope you located the conclusion in the last sentence,

   namely: " etting up a travel company that offers tours to the Andaman Islands is a relatively

   risk-free way to make money." This conclusion is based on a survey by Get Going magazine, which

   found that 75 percent of those who want to visit Andaman Islands would like to do so on a pack-

   age tour, whereas package tour providers at present can provide such tours for only 60 percent of

   those wishing to visit the islands. Here are some assumptions we found:

·         The author assumes that everyone who wishes to visit the Andaman Islands will in fact do so.

   If some of the people who wish to visit the islands won't actually do so, then the present tour

   providers may be more than sufficient to handle the numbers wanting package tours who

   actually will visit the islands.

·         The author also assumes that the survey is not biased. What if Get Going magazine specializes

   in reviewing package tours and its readership is much more likely to prefer package tours than

   your average tourist? If the survey is not representative, then the conclusion is not valid.

2. The Chromium Chimps Argument: Here the conclusion was found in the first sentence: "teady

   chromium intake, even in small doses, can significantly lower the life expectancy of some pri-

   mates”. This conclusion was based on the study in which chimps given small but constant doses

   of chromium in a laboratory had average life spans of 11 years, compared with average life spans

   of chimps in the wild of about 40 years. This is a classic causal argument and here are some

   assumptions we found:

·         The author assumes that chimps raised in a lab and chimps in the wild would normally have

   similar life expectancies.

·         Even more important, the author assumes that there wasn't something else that caused the

   chimps in the study to die prematurely. Who knows? Maybe most of the chimps succumbed

   to a sudden outbreak of Ebola virus. The point is that in a causal argument, it's always

   assumed that there's not another plausible explanation for what happened.

Let's recap what we've learned about the structure of arguments

Evidence + (Assumption) ---> Conclusion

These are the components of an argument. When you break down an argument on the GMAT, you

should begin by identifying the author's conclusion (what the author wants you to believe) and the

evidence (why the author thinks you should accept the conclusion). From there you should be able

to identify the missing steps or gaps between the evidence and the conclusion. These are the author's assumptions.

Spotting the author's assumptions will obviously help you to answer questions that ask about

assumptions. But uncovering an argument's assumptions is also key to handling many other types of

questions. It's time to examine the types of questions that commonly appear on Critical Reasoning.

5#
 楼主| 发表于 2009-9-9 00:37:00 | 只看该作者

COMMON CRITICAL REASONING QUESTION TYPES

Now that you know how to identify the parts of an argument, let's look at how to apply the Kaplan

Four-Step Method to the most common Critical Reasoning question types.

Assumption Questions

Let's take another look at the question with which we began this chapter

1.       A recent study has concluded that, contrary to the claims of those trying to ban cigarette advertisements altogether, cigarette ads placed on billboards and in magazines have little to no effect on the smoking habits of the smokers who view the ads. The study, which surveyed more than 20,000 smokers and solicited their reasons for continuing to smoke, found that practically no one in the survey felt that these advertisements influenced their decision to smoke.

(最近的研究得出,与禁烟广告完全相反,在室外广告栏和杂志上的香烟广告没有影响看广告的吸烟者的吸烟习惯。据2000吸烟者和诱使他们持续吸烟的原因的调查发现几乎没有人在调查中感觉到这些广告影响他们吸烟的决定)

The study's conclusion is based upon which of the following assumptions?

A.       People do not switch cigarette brands based on their exposure to cigarette ads on

     billboards and in magazines.(没有谈到换品牌的问题)

B.       Cigarette ads on billboards and in magazines do not encourage nonsmokers to take

     up the habit.(没有涉及到广告栏的内容问题)

C.       Banning cigarette ads altogether will encourage people to give up smoking.(原理是这样,但是此处没有提到,太简单的道理不可能是答案)

D.  People are consciously aware of all the reasons they choose to smoke.

E.  People who decide to smoke do so for rational reasons.(没有涉及到吸烟人的比例问题)

To answer an Assumption question, begin by identifying the conclusion and evidence in the stimulus, and then try to spot the author's key assumption. This should make it easy for you to locate the correct answer. But in case you run into any trouble, note that wrong answers to Assumption questions

commonly do one of the following:

·          They go beyond the scope of argument. Each argument is written within narrow parameters.   Wrong answer choices will typically go outside the terms of the author's debate.

·          They use language that's too extreme, once again going beyond what the author is attempting  to claim.

·         They do not support the argument. Remember that an assumption must support the

   argument. Even more than that, it rnustbe true for the argument to be valid.

That final point deserves extra emphasis. In fact, to prove that you've actually located the answer

choice that contains the assumption, you can always employ the Denial Test. Here's how it works:

Simply deny or negate the statement and see if the argument falls apart. If it does, you've located a

necessary assumption. If, on the other hand, the argument is unaffected, the choice is wrong.

(加“Not”的方法,在选项中加“not,如果原文的结论不成立,则该选项为正确答案;如果原文的结论没有受到影响,那么该选项不是答案。)

Now let's see if you found the correct answer to the above question.

The argument concludes that cigarette ads placed on billboards and in magazines have little to no

effect on the smoking habits of the smokers who view the ads. This conclusion is based upon survey

of 20,000 smokers which found that practically none of them felt that these ads influenced their

decision to smoke. Choice (D) identifies the key assumption here. If people aren't consciously aware

of all of the reasons they choose to smoke, then the fact that those surveyed don't feel that the ads

influence their decision to smoke does not prove that these ads don't affect their smoking habits.

Now let's look at the other choices:

A.       People do not switch cigarette brands based on their exposure to cigarette ads on

     billboards and in magazines.

This statement goes outside the scope of the argument. The argument concerns smokers' decision to

smoke, not which brands they choose once they've decided to smoke.

B.        Cigarette ads on billboards and in magazines do not encourage nonsmokers to take

     up the habit.

This choice is again outside the scope of the argument. We're concerned with the smoking habits of

smokers who view the ads. How nonsmokers react to the ads is of no concern to us.

C.      Banning cigarette ads altogether will encourage people to give up smoking.

This choice contradicts the argument, which is the opposite of what we're looking for. An assumption

should support the conclusion.

D.       People who decide to smoke do so for rational reasons.

This choice goes too far. There's nothing in the argument that implies that people who decide to

smoke do so for rational reasons.

You'll probably see a couple of Assumption questions in your Verbal Section. Here are some other

ways that an Assumption question could be phrased on the test:

~ Which of the following is assumed by the author?

~  Which of the following, if added to the passage, would make the conclusion valid?

~  The validity of the argument depends on which of the following?

~  The argument presupposes which of the following?

Now let's look at some other questions that also require you to identify the author's assumptions Part Two: Verbal Strategies I

 

Strengthen and Weaken Questions

By far the most common Critical Reasoning question type will ask you either to strengthen or to weaken the argument. About half the questions in your Verbal Section will ask you to do one or the other. Strengthen/Weaken questions are usually pretty self-explanatory; here's how they might be worded:

Strengthen:

    ~  Which of the following, if true, would most strengthen the argument above?

    ~  Which of the following, if true, would provide the most support for the argument above?

     ~  The argument above would be more persuasive if which one of the following were found to be true?

Weaken:

     ~  Which of the following, if true, would most weaken the argument above?

     ~ Which of the following, if true, would most seriously damage the argument above?

     ~  Which of the following, if true, casts the most doubt on the argument above?

When you're asked to strengthen or weaken an argument, once again you want to break apart

the argument by first identifying the evidence and conclusion, and then finding the author's key

assumptions. Here's why:

·         To strengthen an argument, you want to pick the choice that fills in a key assumption

·         To weaken an argument, you want to pick the choice that denies or undercuts a key assumption.

 

 If you ‘re   having trouble spotting a key assumption, look for the anser choice that wither reinforces or undermines (depending upon the question) the validity of the author’s conclusion. it also pays to know that wrong answers to strengthen /weaken questions commonly do one of the following:

 

 

·         They go outside the scope of the argument. Even though these questions ask for new information that, if true, would strengthen or weaken the argument, this information still has to be consistent with the terms and logic of the argument.

·         They introduce irrelevant comparisons.

·         They do the opposite of what they're supposed to-that is, they weaken the argument when they're supposed to strengthen it, or vice versa.

Note, however, that strong wording is not an eliminator on Strengthen/Weaken questions. In fact, if the wording to a Strengthen/Weaken answer choice is too qualified (containing words like some occasionally, or possibly), you should ask yourself whether the statement is truly strong enough to affect the argument.

Try this one out for size. Afterwards you can compare your work to ours  .

2. Whitney Hospital's much publicized increase in emergency room efficiency, which its

spokespeople credit to new procedures for handling trauma patients, does not withstand

careful analysis. The average time before treatment for all patients is nearly 40 minutes--

the highest in the city. And for trauma victims, who are the specific target of the guidelines,

the situation is even worse: The average time before treatment is nearly half an hour--

more than twice the city average.

(W 医院声称增加ER的效率,医院的发言人赞扬新的处理外伤病人的手续,经不住仔细分析。在治疗之前的平均等待时间是在城市最长的几乎40分钟。然而对外伤受害人尤为是这个指导方针的主要目标,
            
这种形式更糟糕:
            
在治疗之前等待时间几乎半个小时是城市平均的2)

Which of the following, if true, would most seriously weaken the author's conclusion

about the value of the new procedures?

A.       The city hospitals with the most efficient emergency rooms utilize the same proce-

     dures for handling trauma patients as does Whitney Hospital.

B.       After the new procedures went into effect, Whitney's average time before treatment

     for trauma patients and patients in general dropped by nearly 35 percent.

C.       Because trauma patients account for a large percentage of emergency room patients,

     procedures that hasten their treatment will likely increase overall emergency room

     efficiency.

D.       Due to differences in location and size of staff, not all emergency rooms can be

     expected to reach similar levels of efficiency.

E.        The recently hired administrators who instituted the new procedures also increased

     Whitney's emergency room staff by nearly 15 percent.

The author concludes that, contrary to publicized reports touting its new procedures for handling trauma patients, Whitney Hospital's ER is no more efficient than before. As evidence the author points out that Whitney's ER waiting periods for patients, and for trauma patients in particular, are the highest in the city. But does this evidence truly support the conclusion? Not really. Just because Whitney's ER is the slowest in the city, that doesn't mean that its efficiency hasn't grown by leaps and bounds. Evidence comparing Whitney to other hospitals (which is all the author presents) is irrelevant. What matters is how Whitney's present efficiency compares to its previous efficiency. If (B) is true, then Whitney's efficiency is up, likely due to the new procedures. Even though it's still the slowest ER around, it's improved markedly.

As for the other choices

A.       The city hospitals with the most efficient emergency rooms utilize the same procedures for handling trauma patients as does Whitney Hospital.

This choice introduces an irrelevant comparison. The fact that these procedures are used in the best

ERs proves nothing. First, the other ERs could be fast for reasons other than the procedures. Second,

even if the procedures generally increase efficiency, Whitney's staff could be so incompetent that the

procedures are ineffective. We need information about Whitney's past ER efficiency, not information

about other hospitals.

B.     

C.       Because trauma patients account for a large percentage of emergency room patients,

     procedures that hasten their treatment will likely increase overall emergency room efficiency.

This choice is too weak to affect the argument; even if the procedures are intended to speed the treatment of trauma victims (that's not actually said, you notice), there's no reason to assume that they worked at Whitney.

D.      Due to differences in location and size of staff, not all emergency rooms can be

    expected to reach similar levels of efficiency.

This might excuse Whitney for offering below-average care, but it gives us no reason to think that

Whitney's care has improved, which is what we need.

E.       The recently hired administrators who instituted the new procedures also increased

     Whitney's emergency room staff by nearly 15 percent.

This choice, if anything, strengthens the conclusion, because it gives us another plausible explanation

for any possible increases in ER efficiency. Even more problematic, this choice provides no reason to

believe that there has been an increase in efficiency, which is what we need to weaken the argument.

Flaw Questions

Flaw questions are less common than Strengthen/Weaken or Assumption questions, but you could

well see one on Test Day. It could be phrased in various ways:

·          The argument above is flawed in that it ignores the possibility that...

·         Which of the following points to the most serious logical flaw in the argument above?

·         Which of the following would reveal most the absurdity of the conclusion drawn above?

Don't confuse these questions with Weaken questions. On a Weaken question, you're supposed to

find additional information that, if true, would weaken the argument, whereas on a Flaw question, you know the evidence doesn't support the conclusion very well at all, and it's up to you to explain why.

You should begin by breaking down the argument as usual, and then pick the answer choice that

explains why the evidence falls short. It should also be fairly easily to eliminate wrong answer choices

on this question type. You can simply get rid of any answer choice that misrepresents the argument.

Try out this strategy here, and then compare your work to ours.

3.  It doesn't surprise me that the critic on our local radio station went off on another tirade

     today about the city men's choir. This is not the first time that he has criticized the choir.

     But this time his criticisms were simply inaccurate and unjustified. For ten minutes, he

     spoke of nothing but the choir's lack of expressiveness. As a professional vocal instructor,

     I have met with these singers individually; I can state with complete confidence that each of

     the members of the choir has quite an expressive voice.  

(我没有感到惊奇,批评家今天在我们当地的电台关于城市男唱诗班进行另外一个长篇激烈的演说。这不是第一次他批评这个唱诗班。但这次他的批评是简单地不准确的和不公正的。10分钟,他除了说唱诗班缺乏表现之外什么都没说;我完全有信心地陈述每一个成员有非常富于表情的声音)

Which of the following indicates the most serious flaw in the author's reasoning?

A.       He directs his argument against the critic's character rather than against his claims.

B.      He ignores evidence that the critic's remarks might in fact be justified.

C.       He cites his own professional expertise as the sole explanation for his defense of the

     choir.

D.       He assumes that a group will have a given attribute if each of its parts has that attribute.

E.        He attempts to conclude the truth of a general situation from evidence about one

     specific situation.

The evidence here is that each member of the choir individually has an expressive voice. But the

conclusion is that the choir as a whole is expressive. But just because each voice is expressive alone

doesn't mean that all the voices will be expressive together. (D) points out this flaw in the author's

reasoning.

As for the wrong answer choices

A.      He directs his argument against the critic's character rather than against his claims.

Although the author is rather vehement in disputing the critic's claims, he doesn't address the critic's

character.

B.       He ignores evidence that the critic's remarks might in fact be justified.

There is no evidence provided in support of the critics remarks; the flaw is not that the author ignores

evidence that might hurt his argument, but that he uses evidence that doesn't really prove anything.

C.       He cites his own professional expertise as the sole explanation for his defense of

     the choir.

This choice is a little tougher because the author does provide his own opinion as his only explanation

Yet he is a professional vocal instructor, and we have no reason to doubt his qualifications.

D.     

E.        He attempts to conclude the truth of a general situation from evidence about one

     specific situation.

This choice doesn't make any sense. There's no attempt made to generalize from a specific situation

here. The author limits his conclusion to attacking, with a specific observation, the radio critic's very

specific critique.

All the Critical Reasoning questions that we've looked at so far have required us to identify the

component parts of an argument. But this is not always the case, as we shall now see.

Inference Questions (without critical reading)

You're likely to see a few Inference questions on the GMAT. On these questions, it's no longer necessary

for you to distinguish between the evidence and the conclusion when you read the stimulus. In fact, if

anything, you can treat the entire stimulus as evidence and draw your own conclusion. There are many

ways that an Inference question could be worded, including the following:

·         The facts above best support which of the following conclusions?(上面支持下面。上à)

·         Which of the following conclusions can be properly drawn from the information above?

·          If the statements above are all true, which of the following can be properly inferred on the basis of them?

·          If the statements above are true, which of the following must be true?

We just noted that an Inference question asks you to treat the entire passage as evidence and then

form your own conclusion. But, as we mentioned in the Reading Comprehension chapter, you are

never supposed to infer too much on the GMAT. In fact, the key to answering an Inference question

is to pick the choice that must be true without bringing in anything from outside of the passage.
            If a

choice needs any assumptions in order to work, then that choice is wrong.

Consequently, wrong answers to an Inference question typically

·         Go outside the scope of the passage

·         Are too extreme (be wary of words such as never, a/ways, must, etc.)

Pay particularly close attention to the wording of the answer choices. Only one choice will be
            
an

absolutely safe deduction from the passage. To see how this works, try answering the following

question. Look for the proper inference, and then compare your work with ours.

4.  When parents allow their children to spend a large amount of time watching television,

     those children see many more images of violence than do children who watch very little

     television. The more violent images a child sees, the more violent that child will become

     The more violent a child is, the more likely the child is to commit crimes as an adult.

If the statements in the passage above are true, which of the following must also be true?

A.       With an increase in the number of acts of violence committed by children, one can

     expect to find a concurrent increase in the amount of television watched by children.

B.       If parents did not allow their children to watch television, juvenile delinquency

     would be unlikely.

C.       No child will develop an aversion to violence if he or she is permitted to watch television,

D.      The more parents try to discourage their children from watching television, the

     more likely those children are to become criminals.

E.       If a child sees more images of violence on television, the likelihood of that child

     committing crimes as an adult increases.  

The point of the passage is that the more television a child sees, the more images of violence he is

likely to see, and thus the greater the chances of his becoming violent and then engaging in criminality

as an adult. So TV viewing is being set up as sufficient to bring about an increase in violence and

criminality. It is not, however, set up as necessary to an increase in violence and criminality. The

argument leaves open the possibility that other factors can effect an increase in violence. Once we

grasp this distinction we can go about eliminating answer choices.

A.       With an increase in the number of acts of violence committed by children, one can

     expect to find a concurrent increase in the amount of television watched by children

This choice infers that with an increase in child violence one should expect an increase in children's

television viewing. Not necessarily. As we just said, other factors could account for an increase in

child violence.

B.       If parents did not allow their children to watch television, juvenile delinquency

     would be unlikely.

This choice infers that delinquency is unlikely if children's TV viewing is prohibited. Not necessarily,

Other factors may also make delinquency likely.

C.       No child will develop an aversion to violence if he or she is permitted to watch
            
television.

This choice is way too strongly worded. The passage merely claims that more TV violence leads

to more violent children, not that no child who is permitted to watch -IV will develop an aversion

to violence.

D.       The more parents try to discourage their children from watching television, the

     more likely those children are to become criminals.

This choice directly contradicts the stimulus. The stimulus says that allowing, not discouraging,

children to view TV will make them more likely to become criminals.

E.        If a child sees more images of violence on television, the likelihood of that child

     committing crimes as an adult increases.

Finally, we have a valid deduction. The stimulus says that children who see more violent images on

TV become more violent, and thus, more likely to commit crimes as adults. This choice, the correct

answer, merely omits the middle term (become more violent) and moves from the violent images

straight to the increased likelihood of criminality.

Note: In this example, the correct answer was the logical conclusion to which the author appeared

to be heading. This will not always be the case. Sometimes an inference will logically complete an

argument; other times, the proper inference may be a trivial point that could easily be dismissed. Just

make sure you pick the one choice that must be true.

Explain Questions
                
 (without critical reading)

"Explain" questions are the final common Critical Reasoning question type. These questions are

different from all the others we've seen; this is because the stimulus to an Explain question does not present an argument at all, but rather describes a situation with two or more seemingly contradictory facts. Your job is to explain away this seeming contradiction. You're likely to see at least one Explain question on Test Day.

Explain questions may be worded many ways, including the following

·         Which of the following, if true, would explain the discrepancy described above?

·         Which of the following, if true, contributes most to an explanation for the apparent

   contradiction noted above?

·         Which of the following, if true, best helps to resolve the paradox outlined above?

These questions ask you to explain how two or more seemingly contradictory facts can simultaneously be true. Before attempting to find an explanation, make sure you have a good grasp of the situation that needs explaining. Only one answer choice will address all of the facts in dispute.

Wrong answer choices, on the other hand, usually:

·         Touch upon only one of the facts  in dispute

·         Deepen the mystery

·         Make an irrelevant comparison

·         Misrepresent the scope of the situation

Give the following a try, and then compare your work to ours

5.   The number of applicants applying to top graduate programs has declined by more than

     10 percent since the mid-1990s. However, the number of students admitted to these same

     programs has not decreased commensurately, and the caliber of the students admitted,

     as measured by their undergraduate GPAs and standardized test scores, has actually

     improved markedly.

     Which of the following, if true, best explains the seemingly contradictory trends described

     above?

A.       The number of applicants applying to second- and third-tier graduate programs has

     also declined by more than 10 percent since the mid-1990s.

B.       Many potential graduate school applicants are put off by the ever-increasing cost of

     a graduate education.

C.       Because of improved economic opportunities, general interest in leaving the work-

     force to go to graduate school has waned in recent years.

D.      The greater premium awarded to degrees from top graduate programs has made

     the applicant pool increasingly competitive, discouraging those with less impressive

     academic records from applying.

E.        The number of women applying to top graduate programs has increased since the

     mid-1990s, while at the same time the number of men applying to top graduate

     programs has declined. Part Two: Verbal Strategies

The passage sets up the following contradictory facts to be resolved:

*The number of applicants applying to top graduate programs has declined by more than

10 percent since the mid-1990s.

*The number of students admitted to these same programs has not decreased  commensurately, and the caliber of the students admitted, as measured by their undergraduate GPAs and standardized test scores, has actually improved markedly.

Choice (D) alone addresses both sides of the contradiction, explaining that these programs have

become increasingly competitive due the higher premium placed on getting a graduate degree from

these institutions, and that this in turn has discouraged those with less impressive records from even

bothering to apply.

Now let's look at the wrong answer choices

A.       The number of applicants applying to second- and third-tier graduate programs has

     also declined by more than 10 percent since the mid-1990s.

This choice is outside the scope of the situation to be addressed. We are not interested in the trends in

graduate programs other than the top ones.

B.       Many potential graduate school applicants are put off by the ever-increasing cost of

     a graduate education.

This choice addresses only one side of the contradiction. The ever-increasing cost of graduate school

may help explain why the number of applicants to top schools has gone down, but it does nothing to

address why the caliber of those applying has nonetheless improved.

C.      Because of improved economic opportunities, general interest in leaving the work-

     force to go to graduate school has waned in recent years.

This choice likewise addresses only the first side of the contradiction. Nowhere does it explain why the

caliber of the remaining applicants has improved.

D.     

E.       The number of women applying to top graduate programs has increased since the

     mid-1990s, while at the same time the number of men applying to top graduate

     programs has declined.

Finally, this choice introduces an irrelevant comparison. The fact that more women and fewer men are

applying to top graduate programs does nothing to address either side of the contradiction.

6#
 楼主| 发表于 2009-9-9 00:40:00 | 只看该作者

12道练习题及答案

PRACTICE QUESTIONS

Now that we've examined how to apply the Kaplan Four-Step Method to all the most common

Critical Reasoning questions, you should be ready to try out what you've learned on the following

practice questions. If you encounter a question type you haven't seen before, fear not. Identifying

the conclusion, evidence, and underlying assumptions of the argument will most likely be the key

to solving the question. If you can't ascertain the correct answer, eliminate as many wrong answer

choices as you can, make your best guess, and move on.

1.   Enrollment in graduate and professional programs tends to be high in a strong economy

     and much lower during recessions. The perceived likelihood of future job availability,

     therefore, affects people's willingness to pass up immediate earning potential in order to

     invest in career-related training.

The argument above assumes that

A.       the perceived likelihood of job availability has decreased in recent years

B.        all those who avoid graduate and professional school during an economic slump do

     so because of the perceived lack of future jobs

C.       perceptions of the likelihood of job availability are related to the state of  the economy

D.       those who enroll in graduate and professional schools during a strong economy help  increase the economy's strength

E.       graduate and professional programs admit fewer students during recessions

 

1.       The university's decision to scale back significantly its teaching of the literary and

philosophical classics of the Western Tradition is misguided. Proponents of the move

argue that today's students are not interested in these works and desire more practical

business-related courses that will help them in their future careers. But any student lacking

a sufficient grounding in the thought and tradition that underlie the present civilization

cannot be said to be fully educated. The classics are the primary vehicle for instilling such

knowledge.

Which of the following best expresses the relationship between the two bolded statements

above?

A.       The first statement offers a hypothesis and the second statement offers conflicting

     evidence.

B.       The first statement suggests an alternative explanation for the phenomenon

     described in the second statement.

C.      The second statement provides evidence for a conclusion drawn in the first

statement.

D.      The second statement must be true for the first statement to be true.

E.       The second statement is an inference drawn from the first statement.  

2.        Due[MSOffice1]  to a string of dismal performances, a touring band has begun to lose its audience.

     News of the disappointing concerts has traveled quickly via cable stations and the Internet,

     and has negatively influenced ticket sales for future performances. Due to the poor ticket

     sales, a number of promoters have canceled the band's upcoming shows, forcing the band

     to attempt to recoup its touring and recording expenses from fewer total performances.

Which of the following, if true, taken together with the information above, best supports

the prediction that more of the band's shows will be canceled?

A.        The promoters who canceled shows did so with the promise that they would

     monitor the band's reception in other cities before deciding whether or not to

     reschedule the canceled shows.

B.      The pressure to restore its diminishing fan base and recoup its overall expenses

     from a decreased number of performing opportunities is likely to cause the band to

     perform poorly in future concerts.

C.       Because of the canceled shows, it will be impossible for the band to earn a profit on

     the current tour.

D.       If the band cannot salvage the tour, its next CD will likely fail economically unless

     the band can restore its image through music videos.

E.        It is impossible for the management of a rock band to predict accurately the success

     of a tour because fans of rock bands are notoriously fickle in their tastes.

3.       Over the past several years, Running River Water Park has experienced a serious decline in

attendance and sales despite the addition of several state-of-the-art water slides. This year,

the Board of Directors lowered the park's weekday admission prices in order to attract

more customers. Attendance during the first two months of this year's season has been

30 percent higher than the attendance during the same two months last year. Clearly, the

price cut has had the desired effect.

Each of the following, if true, weakens the conclusion above EXCEPT

A.       Nationwide, the number of people attending amusement parks has increased by

     30 percent this year over last year.

B.       Grand Excursions Amusement Park, located ten miles from Running River, has been

     closed during most of this year's season due to unexpected equipment problems.

C.      The most popular movie released this summer, The Big Chase, features a long action

     sequence that was filmed at Running River.

D.      Several large businesses relocated near Running River during the past year bringing

     with them many employees and their families.

E.        Most amusement park visitors are aware of the admission prices before they arrive at

     an amusement park.   

4.       Staff[MSOffice2]  members at the Willard Detention Center typically oversee students' schedules and

make all final decisions regarding the required activities in which students participate.

Students are permitted, however, to make their own decisions regarding how they spend

their free time. Therefore, students should be permitted to make their own decisions

regarding the elective courses that they wish to take.

The conclusion above would be more reasonably drawn if which of the following were

inserted into the argument as an additional premise?

A.        Decisions regarding required activities are more important than decisions regarding

     the elective courses that students take.

B.       Students are more willing to take elective courses than to participate in required Center activities

C.      Required activities contribute more to the students' rehabilitation than do their free-time activities.

D.      Staff members at Willard have found that elective courses are more beneficial for students than the available free-time activities.

E.       When compared for decision-making purposes, elective courses are more like free-

     time activities than required activities.

6.   The average math score on a state-wide proficiency exam for students attending

     Middlebury High School last year was 20 points higher than the average math score for

     students attending nearby Ellingsford High School. Therefore, any student at Ellingsford

     High School wishing to achieve a better math score on next September's proficiency exam

     should transfer to Middlebury High School over the summer.

Which of the following statements, if true, would most significantly strengthen the

conclusion drawn in the passage?

A.      Middlebury High School offers its students a unique, week-long course just before

     they take the proficiency exam that has consistently proven effective in raising

     student scores.

B.      One third of all the students who have transferred to Middlebury High School the

     summer before taking the test got scores that are at least 20 points higher than the

     average score at Ellingsford High School.

C.       Middlebury High School students who transfer to Ellingsford High School in the

     summer before they take the proficiency exam get average scores that are comparable to the average scores of students who remain at Middlebury.

D.       In the past five years, the average score at Ellingsford High School has been rising at

     a faster rate than has the average score at Middlebury High School.

E.       Students wanting better proficiency-exam scores are transferring to Middlebury

     High School at a high rate, which will ultimately result in a lowering of the school's

     average score.

7.   A study found that last year roughly 6,000 homeless people in the United States were

     admitted to hospitals because of malnutrition. In the same year, a little more than 10,000

     nonhomeless people were admitted to hospitals for the same reason. These findings

     clearly show that the nonhomeless are more likely to suffer from malnutrition than are the

     homeless.

The answer to which of the following questions would be most likely to point out the

illogical nature of the conclusion drawn above?

A.      What is the relative level of severity of the malnutrition suffered by each group cited

     in the study?

B.       To what extent, on average, are the nonhomeless better off financially than the

     homeless?

C.       To what extent are the causes of malnutrition in the nonhomeless related to igno-

     rance of proper dietary habits?

D.       What percentage of each group cited in the study suffered from malnutrition

     last year?

E.        What effect would a large increase in the number of homeless shelters have on the

     incidence of malnutrition among the homeless?

8.  History[MSOffice3]  has shown that severe and sudden political instability strikes the Republic of

Balanda roughly once every 50 years. The most recent example was the attempt on the

president's life in 1992. The reaction of average investors in Balanda to crisis situations in

the country cannot be predicted in advance. The government's fiscal affairs department

has introduced an electronic protection mechanism into the market in the hopes of

avoiding a prolonged large-scale selloff. The mechanism is triggered in specific instances

based on estimations of how average investors will react to changes in corporate data and

economic indicators.

If the statements above are true, which of the following conclusions can be drawn

regarding the electronic protection mechanism?

A.      Sometime within the next 50 years severe and sudden political instability in Balanda

     will trigger the protection mechanism.

B.       Whether the protection mechanism will function appropriately in response to a

     sudden political event depends on whether the event is seen by investors as positive

     or negative.

C.       It is unclear how well the protection mechanism would work in the event of a

     sudden political coup if such an event were partially or wholly unrelated to changes

     in corporate data and economic indicators.

D.      There would be no way for the protection mechanism to differentiate between

     market fluctuations resulting from economic factors and those that are caused by

     political instability.

E.        The protection mechanism would be purposely destroyed by political insurgents if

     they were able to infiltrate the government's fiscal affairs department.

 

9. Country X complains that Country Y's high tariffs on imported goods have artificially

inflated the price of cars imported from Country X into Country Y, and that this is the

reason that few of Country X's cars are sold in Country Y. On the other hand, Country X's

very low tariffs allow Country Y to sell many cars there at relatively low prices. Country X

says that if Country Y would lower its tariffs, then Country X's cars would be able to

compete in Country Y and an equitable balance of trade would be achieved.

Which of the following, if true, would most undermine the validity of Country X's

explanation for the poor sales of its cars in Country Y?

A.       In places where the tariffs on goods from both countries are equal, Country Y's cars

     far outsell cars from Country X.

B.       Cars imported from Country Z sell poorly in Country Y.

C.      In countries where tariffs on imported goods are higher than in Country Y,

     Country X sells more cars than does Country Y.

D.       Other goods from Country X sell poorly in Country Y.

E.       Sales of Country Y's cars are high even in countries that have higher tariffs on

     imported goods than does Country X.

10.  Many adults, no matter what their age, respond to adversity by seeking advice only from

     their parents. Consciously or not, they regress to a psychological state of childhood

     dependence in which the parent is seen as the only source of wisdom and comfort. Adults

     who do not regress to this childhood mode turn for advice in adversity only to other loved

     ones--a spouse or a best friend  whom they perceive and relate to as peers.

If all of the above statements are true, which of the following must also be true?

A.       One's parents offer more wisdom in adversity than those whom one perceives

     as peers.

B.       Adults who do not suffer adversity look only to their parents for advice.

C.       No adults seek advice in adversity from total strangers.

D.       Adults who seek advice in adversity from their parents do not expect to receive

     wisdom and comfort.

E.       Adults who regress to a state of childhood dependence lose touch with their peers

11. Recent experiments in the Southern Ocean offer the promise of controlling the threat

of global warming by creating organic "sponges" for carbon dioxide, which is widely

considered the main culprit for rising global temperatures. Scientists were able to grow

a lush strip of phytoplankton 150 kilometers long by fertilizing a patch of the ocean with

hundreds of kilograms of an iron compound. Phytoplankton presently accounts for over

half of the photosynthesis on Earth, the process by which carbon dioxide is absorbed and

converted into oxygen. A major limiting factor in the production of phytoplankton is lack

of iron, but by fertilizing oceans with iron compounds scientists hope to be able to reduce

carbon dioxide levels and reverse the greenhouse effect .

Which of the following, if true, casts the most serious doubt upon the advisability of using

the fertilization method described above to control carbon dioxide levels?

A.       In some oceans, the growth of phytoplankton is also limited by how much nitrogen,

     phosphorus, and silicon are available.

B.      The cost of fertilizing the oceans with sufficient quantities of iron to reverse the

     greenhouse effect is likely to be very high.

C.      Iron naturally reaches the seas in the form of wind-blown mineral dust, which

     becomes more or less abundant as conditions on land change.

D.       Fertilization efforts will do nothing to curb the production of more carbon dioxide

     emissions.

E.       The oceans are a complex system, and the long-term consequences of ocean fertiliza

     tion may be the opposite of what is predicted.

12.  University systems that use graduation rates to determine which campuses are allotted

     additional funds are acting counter to their stated goals. The universities say they are

     trying to raise academic standards, yet they are actually encouraging campuses to graduate

     students regardless of achievement.

Which of the following statements, if true, would help to validate the approach taken by

the university systems mentioned above?

A.       Graduation rates for university systems with this policy are among the highest in the

     nation, but the graduates from these systems score poorly on tests of basic skills.

B.      The campuses that need the additional funds the most are the ones that have the

     lowest graduation rates.

C.       The new funds will be allotted for facility upgrades, not new faculty positions or staff

     pay mcreases.

D.       Graduation examinations currently exist that require every graduate at each

     university to demonstrate a minimum level of achievement.

E.       An opposing plan focuses on providing extra funding based not on graduation rates,

     but on the percent of students that pass basic skills tests. ANSWERS AND EXPLANATIONS

 

 

 

ANSWERS AND EXPLANATIONS

1.  Choice (C) is correct.

We're asked to find an assumption. The argument involves a question of cause and effect. Since

enrollment in graduate and professional programs tends to be high when the economy is strong and

low when it is weak, the reason must be, according to the author, a matter of people's perceptions of

job availability. Sounds reasonable, but do all the terms match up with those in this conclusion? We

know from the question stem that they do not. What's missing? Well, the evidence pertains to the

state of the economy. But the conclusion strays into the area of psychology-people's perceptions. Are

these the same things? The author treats them as such by arguing from evidence regarding the state

of the economy to a conclusion based on people's perceptions of the economy. The author takes the

relationship between these for granted, but technically, in order for the argument to work, this must be

established. (C) reveals this basic assumption.

2.  Choice (0 is correct

Questions such as this one, which ask about the functions played by the bolded sentences in an

argument, have become increasingly popular on the GMAT-so there's a good chance you'll see one

on Test Day. Once again, your primary task is to break down the argument and identify its parts. Here

the conclusion is in fact the first sentence of the passage; the rest of the passage goes on to explain

why the university's decision is misguided. The first sentence, when it is phrased as an assertion,

as here, is often the conclusion of the passage. So why is the university's decision misguided?

(1) Students lacking a certain grounding cannot be considered fully educated. (2) The classics are

the primary vehicle for instilling such knowledge. So the final sentence (second statement) provides

evidence for the conclusion drawn in the first sentence (first statement), which is exactly what

(C) says. You can also use the process of elimination. (A) is out because the second statement offers

no conflicting evidence. (B) is out because the second statement does not describe a phenomenon

and the first statement suggests no alternative explanation. (D) is out because the second statement

does not have to be true in order for the first statement to be true, nor is the second statement an

inference, or logically drawn conclusion, of the first statement, as (E) states.

3.  Choice (B) is correct.

This question is a little unusual in that the conclusion-more of the band's shows will be canceled-

appears in the actual question. You already know the argument's conclusion, so read the passage

carefully and follow the flow of the evidence presented. Dismal performances led to a drop in ticket

sales for future concerts, which in turn led concert promoters to cancel concerts, putting additional

pressure on the band to make up the lost money from fewer shows. We're looking for the choice

that leads to more canceled shows, and it's not easy to predict the answer here. But you can test

each choice to see where it fits into this chain of events, if at all. Canceled concerts must now lead

to a result that will trigger additional canceled concerts. What do you already know leads to canceled

concerts? Dismal performances and lower ticket sales. Keep this in mind while testing out the choices

(B) works by turning the chain of events into a vicious cycle. If (B) were true, we would expect.

better math scores should transfer to Middlebury the summer before they take the test. Think about

the time frame employed here: The author assumes that former Ellingsford High School students

will learn what they need to improve their math scores almost immediately after transferring to

Middlebury. The correct answer will most likely offer a plausible reason why this might not be the case.

(A) provides a plausible reason to expect that students who transfer to Middlebury during the summer

can significantly benefit before taking the test in September by taking advantage of the unique, week-

long course offered just before test day. If (A) is true, then a major stumbling block to the potential

efficacy of the proposal is removed, and the argument would be strengthened.

As to the wrong choices: (B) tells us nothing. The average score of only part of a group cannot be

fairly compared to the average score of an entire group. A third of the students who remained at

EIlingsford High School may also have scored at least 20 points higher than their school's average.

(C) weakens the argument's conclusion. If Middlebury students who transfer to Ellingsford during the

summer before they take the test still get higher math scores, then the notion that transferring the

summer before is a cure-all seems less plausible. (D) presents an irrelevant comparison. The scores

at Ellingsford High School may be rising faster than those at Middlebury High School, but the average

score at Middlebury is still 20 points higher than the average score at Ellingsford. (E) goes beyond

the scope of the argument because the argument does not concern itself with what will happen

ultimately. The main issue is whether Ellingsford students can raise their scores by transferring. Even

if (E) is true, and many students transfer and Middlebury's test score average drops, it's impossible

for us to tell whether the transferees' scores are higher or lower than they would have been had they

remained at Ellingsford.

7.  Choice (D) is correct.

The question stem reveals that the argument in the stimulus is fatally flawed. As you read the passage,

identify the evidence and conclusion, and watch for a questionable assumption or some other flaw

along the way. The argument is based on numbers: Since only 6,000 homeless people suffering from

malnutrition were admitted to U.S. hospitals last year, compared to 10,000 nonhomeless people, the

nonhomeless must be more likely to suffer from malnutrition. But total numbers mean nothing! We

cannot figure the odds of suffering from malnutrition solely from the number of malnourished people

in each group. We need to know the percentage of homeless people suffering from malnutrition

versus the percentage of nonhomeless people suffering from malnutrition. Clearly there are far

more nonhomeless people than homeless people in the United States, so the argument contains a

ludicrous assumption: that the two groups are comparably sized. The correct answer to the question

will somehow point this out. (D) provides the question whose answer would provide the information

we need to correctly understand the odds. It allows us to see how the raw numbers cited do not

support the author's counterintuitive conclusion that the nonhomeless are more susceptible to

malnutrition than are the homeless. As to the wrong answers, (A) goes beyond the scope of the

argument. The argument involves the likelihood of suffering from malnutrition, not the relative levels

of severity. (B) also introduces a new issue-finances. No matter how much people with homes are

better off financially than the homeless, the fact remains that more nonhomeless were hospitalized

for malnutrition than homeless. ((2) introduces another new issue. The argument draws no conclusion

about the causes of malnutrition within these groups, only about the likelihood of malnutrition. (E) is

irrelevant to the argument as presented: The future possibility of remedying homelessness to some

degree does not impact upon these numbers and this particular conclusion drawn from them.  

8.  Choice (C) is correct.

You must draw an inference in this question-so you are looking for a softly worded answer that must

be true given the statements in the argument. It's difficult to predict the correct answer here, so your

best bet is to test the choices rigorously, looking for the one that absolutely must be true. (C) draws

a reasonable conclusion based on the evidence. If political instability involves changes in corporate

data and economic indicators, then the mechanism should work the way it is designed to work. But if

the incident does not involve those elements, then how the mechanism will work becomes unclear,

because the behavior of investors will be unpredictable. Of the wrong answers, (A) goes too far when

it infers that a severe and sudden political instability will occur within 50 years. The 1992 attempt was

only an example of the political instability that occurs roughly every 50 years, and the 50-year period

was an average, not an absolute limit. Furthermore, even if there is an attempt on the president's life,

it is unclear how investors will react because their behavior in such situations cannot be predicted in

advance. (B) goes beyond the scope of the argument. Whether investors perceive sudden political

events positively or negatively isn't mentioned in the stimulus, so we can't infer that that perception

makes any difference to the accuracy of the mechanism. (D) takes its inference too far. The

mechanism might be able to differentiate between various types of market fluctuations, even though

it might not be able to trigger appropriate responses to some of them. And (E) takes the argument far

beyond its original scope. Nothing in the stimulus leads to a ~rediction of what might happen to the

protection mechanism in the event of political instability.

9.  Choice (A) is correct.

Find the author's conclusion and evidence, and the assumption linking them. Weaken the argument

by invalidating that assumption. The argument develops from evidence that Y's high tariffs make X's

cars relatively expensive, but X's low tariffs make Y's cars relatively cheap. The argument concludes that

if Y lowers its tariffs, then real competition and an equitable balance of trade would result. Country X

assumes that Y sells more cars in X than X sells in Y because of the price difference created by the

different tariffs. This suggests a classic case of ignored alternatives. Look for a choice that suggests

that there might be an alternative explanation besides the difference in tariffs that could explain the

weakness of X's car sales relative to Y's. (A) weakens count~/X's argument by illustrating that the effect

(more Y cars sold than X cars) is possible without the supposed cause (a tariff imbalance favoring Y).

This implies that something other than tariffs-the reason offered by X for its poor performance in the

car market-could very well explain the difference in sales (maybe X's car's are no good).

10. Choice (C) is correct.

The question stem sends us searching for a valid inference that can be drawn from the statements

in the passage. You're looking for a conclusion that must be true based upon the statements in this

argument. Reading the passage, you learn that adults can be divided into two groups: adults who

seek advice in adversity only from their parents, and the remainder, who seek that advice only from

other loved ones. There is not much room here for argument. All adults fall into one or the other of

these categories, but not both. If these statements are true, then adults must turn either to parents

or to other loved ones in times of adversity, and to no one else. (C) is a valid inference, given the

evidence above. Adults seek advice from parents or other loved ones-not from strangers. Of the

wrong answers, (A) introduces a concept foreign to the evidence. The passage tells us nothing about

the relative wisdom of parents and peers. (For all we know, everyone gives terrible advice.) (B)

introduces another concept foreign to the evidence. The passage tells us nothing about what people

do for advice when they are not in adversity. (D) goes beyond the scope of the evidence. Expectations

are not discussed in the passage. The issue here is where people in trouble go for advice-not what Part  

they expect to find. And (E) goes too far. Adults who regress will seek advice from parents if they face

adverse circumstances and wish to get advice. This doesn't mean they will necessarily lose touch with

peers.

1 I. Choice (E) is correct.

We're looking for the answer that does the best job at weakening the argument. This is a causal

argument, which concludes that a plan to fertilize the oceans with iron compounds will be able to

reverse the greenhouse effect. The chain of reasoning goes like this: By fertilizing the oceans to iron

compounds, phytoplankton will grow, creating organic "sponges" to soak up carbon dioxide, widely

believed to be responsible for global warming. We're looking for an answer that weakens this chain of

causality. (E), which notes oceans are a complex system, and consequences of fertilization could be

the opposite of what's predicted, does a good job of this. What if the plan ends up exacerbating the

greenhouse effect? That would certainly cast doubt upon the advisability of the plan!

Of the wrong answers, (A), which discusses some oceans, is too weak to affect the argument. There's

nothing in the argument to suggest that all the oceans need to be fertilized. (B), which mentions the

high cost of the plan, is beyond the scope of the argument. The argument does not suggest the cost is

a factor; after all, we're discussing a plan to save the planet! (C) is completely irrelevant to the issue at

hand. Who cares about how iron naturally reaches the sea? And finally, (D) does nothing to invalidate

the plan. Sure, emission-reduction plans may also be in order, but that doesn't mean we shouldn't try

to reduce present carbon dioxide levels.

12. Choice (D) is correct.

The trickiest part of this question is understanding whose position we're trying to validate and whose

we're trying to attack (it gets confusing when the argument involves opposing positions, as here).

Here we're trying to validate the approach taken by the university systems, which the author argues

is flawed; so, in other words, we're trying to weaken the author's critique. So what is the approach

taken by the university systems? And what is the author's critique? The systems use graduation rates to

determine which universities are awarded extra funds. This, the author argues, encourages campuses

to graduate everyone, regardless of achievement. (D), which notes that exams exist that require every

graduate to prove a minimum level of achievement, attacks the author's contention that the approach

will lead to people graduating regardless of achievement.

Of the wrong answers, (A) strengthens the author's argument, as it agrees with his contention that

achievement standards are being disregarded. (B) argues against the university systems' approach,

although not for the same reasons as the author. (C) makes a completely irrelevant point. What's at

issue is not how the money is spent by schools, but how it is allottedto them. And (E) doesn't validate

the approach taken by the university systems, but in fact hints that there may be a fairer approach

out there  .

 
7#
 楼主| 发表于 2009-9-9 00:44:00 | 只看该作者

Kaplan 语文部分逻辑套题1

Critical Reasoning practice set one

 

Time: 25 Minutes~16 Questions

Directions: For each question in this section,select the best of the answer choices given.

1.      One problem with labor unions today is that their top staffs consist of college-trained lawyers, economists, and labor relations experts who cannot understand the concerns of real workers. One goal of union reform movements should be to build staffs out of workers who have come up from the ranks of the industry involved.

The argument above depends primarily on which of the following assumptions?

A.       Higher education lessens people's identification with their class background.

B.       Union staffs should include more people with firsthand industrial supervisory experience.

C.       People who have worked in a given industry can understand the concerns of workers in

     that industry.

D.     Most labor unions today do not fairly repr sent workers' interests.

E.       A goal of union reform movemems should be to make unions more democratic.

 

2.      Opening a plant in war-torn Country X is not inadvisable, despite what critics of the plan may say. Ten years ago we opened our plant in Country Y in the middle of a revolution; that plant has been generating substantial profits ever since.

Which of the following is the author of the argument above most reasonably intending the

reader to conclude?

A.       Wars are profitable for the author's particular   business.

B.       Country X is a more politically stable nation than is Country Y.

C.       Critics of the proposed plant in Country X are likely to be biased.

D.     The proposed plant in Country X will generate profits in spite of its present war.

E.      The proposed plant in Country X will be  more successful than the plant in Country Y

3.      Since the deregulation of the air travel indnstry in the late 1970s, air fares have been relatively low and the number of passenger miles flown has been increasing. In recent months, however, air fares have risen but the number of passenger miles flowu is still going up.

Which of the following can most reasonably be inferred from the statements above?

A.       The cost of air travel does not affect the number of passenger miles flown.

B.       People are now flying to destinations that  they formerly reached by train or by other

     modes of transportation.

C.       Factors other than low air fares must be contributing to the rise in the number of

     passenger miles flown.

D.     Takeovers in the deregulated air travel  industry have been responsible for the rise in

     air fares.

E.       Air fares can climb even higher without causing a drop in the number of passenger

     miles flown.         

4.   One Zydol capsule contains twice the pain reliever found in regular aspirin. A consumer will have to take two aspirin in order to get the relief provided by one Zydol. And since a bottle of Zydol costs the same as a bottle of regular aspirin, consumers can be expected to switch to Zydol.

Which of the following, if true, would most weaken the argument that consumers will be discontinuing the use of regular aspirin and switching to Zydol?

 A . regular bottle of aspirin contains more than  twice as many capsules as does a bottle of Zydol.

B.The pain reliever in Zydol is essentially the sanre pain reliever found in regular aspirin.

C. Some headache sufferers experience a brief period of dizziness shortly after taking Zydol

     but not after taking regular aspirin.

D. Neither regular aspirin nor Zydol is as effec- tive in the relief of serious pain as are drugs

     available only by prescription.

E. Zydol capsule is twice as large as the average aspirin.

       5. At a certain college, graduate teaching assistants conduct discussion sections but have no input into grading. It has been suggested that graduate  assistants be given some grading responsibility,  but many undergraduates oppose that proposal. They argue that if grades are assigned by graduate      assistants, regular full-time faculty will devote less  time and attention to undergraduate work.

The information in the passage above answers which one of the following questions?

A.       Are grades assigned by graduate teaching      assistants inherently as fair as those given by regular faculty?

B.       Are some undergraduates in favor of main  the full-time faculty's interest in their

     schoolwork?

C.       May regular full-time faculty conduct discussion sections at the college?

D.      Does graduate student contact with undergraduates' work make the grades assigned by

     regular faculty less valid?

E.       Are regular faculty members in favor of giving      graduate assistants some teaching responsibility?

6.   Air travel is becoming increasingly more dangerous. In the last year there have been seven

     major collisions resulting in over 700 deaths, more deaths than in any previous year.

     Which statement, if true, would most weaken the argument above?

A.      Since the volume of air traffic has been  increasing all the time, an increase in the

     number of deaths due to collisions does not Necessarily mean greater danger

B.       The increase in collisions can be explained  by statistical coincidence, hijackings, and

     unusual weather.

C.       Mortality per passenger mile is lower for air travel than for any kind of surface

     transportation.

D.      The increase in deaths due to collision in  air travel has proceeded at a rate identical to

     that for deaths in all other major forms of   transportation.

E.   Last year the average number of passengers per flown plane was significantly lower than that of previous years.

7.   According to a recent study, attending a single-seX  high school aids an adolescent's physical growth. Cited as evidence is the finding that during the fi two years of high school, the average boy in an al  boys school grew five inches, and the average girl an all-girls school grew four inches.

The answer to which of the following questions Is needed in order to evaluate the reasoning

presented in the study?

A.      Why was it that the first two years of high      school were chosen as the focus of the stud

B.       Did some of the boys in the study grow less th      five inches while they were hi tfigh school?

C.       How much do the average male student and th average female student in a co-educational  row during their first two years of high school

D.      Did the girls in the study have as nutritious      diet as the boys during the time the study

     being conducted?

E.      What was the average height of the boys an   the average height of the girls upon enterin  high school?
            

8.      Although most people know that exercise is good for the body, few realize the extent to which it is valuable to the mind. The blood circulates more rapidly after physical exertion, thus allowing all of the body's organs to operate more efficiently. This increased activity enables the brain to receive more oxygen, thereby creating a higher capacity for concentration.

The main point in the argument above is that

A.       the greater the amount of oxygen the brain receives, the better the brain functions.

B.       exercise is a mental, as well as physical, activity

C.       exercise helps the brain more than it does the rest of the body.

D.      people can greatly improve their powers of concentration by exercising more often.

E.       exercise serves more than one purpose.

9.      Ernesto: Sales of VCRs--videocassette recorders---- will decline in the next few years  because the saturation level among U.S. households   has virtually been reached.

Milton: Everyyear a greater number of popular feature films is released on cassette--at

least six per month. Clearly VCR sales will  Remain constant, if not rise,

Which of the following is the best logical evaluation Of Milton's response to Ernesto's argument?

A.       He cites evidence that, if true, disproves the  evidence cited by Ernesto in drawing his

     conclusion.

B.       He points out a gap in the logic followed by      Ernesto in drawing his condusion.

C.       He cites an issue ignored by Ernesto and that outweighs the issues raised by Ernesto.

D.      He does not speak to Ernesto's point because   he fails to raise the issue of whether VCR

     sales may be linked to sales of other leisure related products.

E.       He fails to respond to Ernesto's argument because he assumes that nothing will signifi-

     cantly retard the sale of VCRs, which was the  issue that Ernesto raised.

 

10.  Archaeologists have discovered various paintings on tire walls and ceiling of a Chinese cave whose entrance was blocked by a volcanic eruption in    the 25th century B.c. and only recently cleared by    an earthquake. Since the paintings depict warriors    using Type C bronze weapons, these[MSOffice1]  archaeologists      have concluded that Type C bronze weapons were      already widely used in this area by 2500 u.c., far earlier than was previously believed.

Which of the following pieces of additional Evidence would most seriously weaken the

archaeologists' conclusion?

A.       Another entrance to the cave remained clear until a second volcanic eruption 1,000 years  after the first.

B.       Archaeologists have evidence that Type C      bronze weapons were in wide use in areas of    present-day India as early as 2500 ~.c.

C.      Ahernative methods of dating place the time of the volcanic eruption somewhat earlier, at   around 3000 B.(.

D.      Most experts believe that Type C bronz weapons were not in use anywhere in

present-day China until 2000 B.c.

E.       The paintings were very faded when the archaeologists found them, making identifi

     cation of the depicted weapons difficult

11.  Considering[MSOffice2]  the current economy, the      introduction of a new brand of cereal is unlikely

     to expand total sales of cereal, but rather will just cause some existing buyers of cereal to switch  brands. So it makes no sense for the Coolidge  Corporation to introduce another brand of cereal, since they will only hurt sales of the brands of   cereal they already produce.

Which of the following, if true, would most seriously weaken the argument above?

A.       Total sales of cereal will increase as the total population increases.

B.       Many new brands of cereal sell extremely well for the first year of their existence.

C.       Coolidge Corporation currently produces fewer brands of cereal than its competitors do.

D.      Some cereal buyers regularly switch from brand to brand, even when no new brands

     have been introduced.

E.       Research indicates that the new brand will attract more buyers of competitors' cereals

     than buyers of other Coolidge brands.

12. A public health official reported that 60 percent of  the children at summer school have never had the measles or chicken pox, and that of this 60 percent not one child has ever been observed to eat the cheese served in the school lunches. From this he conducted that children who abstain from cheese products protect themselves from most childhood disease.

Each of the following, if true, would strengthen the official's argument EXCEPT[MSOffice3] :

A.       Medicany speaking, whatever serves to inhibit measles and chicken pox will generally inhibit   the entire spectrum of childhood diseases.

B. The observations the official carried out were      extremely accurate, and all those observed

     to abstain from cheese at school did, in fact,  abstain.

C. The children's eating habits are the same at school as anywhere else, and those who

     abstain from cheese products at school do so  in general.

D. Recent research has pointed to a deficiency in cheese products as one of the major causes of

     measles and chicken pox infections.

EMost cheeses and cheese products harbor bacteria that are known to be causative agents

     for many childhood diseases, such as measles      and chicken pox.
        

13. A confidential survey revealed that 75 percent of the employees of Company P are dissatisfied with their jobs. However, an investigation into working conditions at the company showed nothing uncommonly bad. Therefore, Company P's consulting firm concluded that the employees' dissatisfaction must result from an unusually high incidence of psychological problems on their part. Each of the following, if true, casts doubt on the

consulting llrm's conclusion EXCEPT:

A.       In the investigation of working conditions,  no account was taken of the fact that for

the past year many Company P employees worked on a joint venture with Company O

at Company O's facilities.

B.       Workers in many companies are dissatisfiedalthough there are no apparent problems

     with their working conditions.

C. The consulting firm's conception of what constitutes uncommonly bad working condi

     tions is not identical to that of Company P's      employees.

D. The reasons given by Company P's employees  for their dissatisfaction varied greatly from

     employee to employee.

E. A battery of tests performed on Company P's employees one month ago revealed no

     significant psychological stresses or problems

 

14.  Director: Our engineers are considering two   different sites--one on the Abaco River

and one on the Bornos River  for a hydroelectric plant. Although we haveThe technical expertise to build roughly the same plant in either place, producing roughly the same amount of electricity per hour, building the plant on the Abaco site will cost over twice as much money.

With our budget currently in deficit, we should build the dam at the Bornos site.

Which of the following, if true, best explains the difference in building costs for the two proposed dam sites?

A.      Many farms along the lower Bornos River  valley would benefit from the controlled flow

     of water a dam would make possible.

B.      The Abaco site is in an inaccessible area, requiring the building of new roads and the

     importation of laborers.

C.      The Bornos site is near a large city whose residents could use the resulting lake for

     inexpensive recreation.

D.      The Abaco site is in an area that contains many endangered species that would be

threatened by the new dam.

E.       The Abaco River has a relatively low volume of flow, making it impossible to expand an

     Abaco plant to meet future electricity needs.            

15. In 1988, 50 people with emotional disturbances underwent hypnosis to be cured of their mood swings. A follow-up survey in 1993 revealed that  had fairly stable emotional conditions at the time of the survey. These 5 subjects can therefore serve as models of the types of people for whom  hypnosis is likely to be successful.

Which of the following, if true, casts the most Doubt on the suitability of those five subjects as

models in the sense describe@.

A.       The five subjects have very different personalities and backgrounds.

B.       Since 1988, the five subjects have experienced dramatic mood swings interspersed with

     periods of relative stability.

C.      Those people who were still suffering from unstable emotional conditions at the time of

the 1993 survey had shown no improvement  since 1988.

D.      Many psychologists are less concerned about a patient's mood swings than about the

     patient's willingness to express his or her problems and fears.

E.       The emotional condition of most of the  45 subjects who were still unstable at the time

     of the 1993 survey had actually worsened since 1988.

16.  The cause of the peculiar columnar growth pattern displayed by junipers growing near

     burning underground veins of lignite coal has never been convincingly explained. Until recent

     the accepted theory posited that the abundance of carbon monoxide in the local atmosphere caused the columnar growth. However, a new      theory holds that the cause is the persistent heat   present near these underground fires that, whill      not intense enough to inflame the trees, can  nonetheless change their normal growth patter

The existence of which of the following would provide the strongest support for the new theoJ

A.       A columnar juniper growing in an atmosphere of intense beat and an absence of

     carbon monoxide

B.       A normal juniper growing in an atmosph  of intense heat and an absence of carbon

monoxide

C.       A columnar juniper growing in an atmosphere of normal heat and a high

     concentration of carbon monoxide

D.      A normal juniper growing in an atnmspb  of intense heat and a high concentration carbon monoxide

E.       A columnar juniper growing in an  atmosphere of intense heat and a high

     concentration of carbon monoxide

 ANSWERS AND EXPLANATIONS

     1.  C

    2.  D

    5.  C

    4.  A

    5.  B

    6.  A

    7.  C

    8.  E

    9.  E

   ]O. A

   ]1. E

   12. D

  73.  D

  14.  B

  15.  B

  16.  A

 

           

8#
 楼主| 发表于 2009-9-9 00:45:00 | 只看该作者

建议大家可以共同讨论,

Kaplan 语文部分逻辑套题1的答案  

 art Three: Practice Sets

Critical Reasoning Practice Set One    161

The Conclusion: Union reform movements should build

staffs out of workers who have come up the ranks.

The Evidence: Union movements are currently suffering

from a problem: Their staffs consist of college-educated

professional types who don't understand the concerns of

the worker.

If the author believes that hiring up-from-the-ranks workers

(an idea introduced in the conclusion) will cure that

problem, he must be assuming that these former workers

do understand workers' real concerns.

There's no need to assume that higher education lessens

people's identification with their class background, (A),

since the author hasn't said that the lawyers, economists,

and experts who don't understand workers come from a

working-class background. Supervisory experience, (B), isn't

the same as coming up through the ranks. Labor unions

having problems, which the author admits, isn't the same as

(D), most of them unfairly representing workers' interests.

That's an overstatement. "Democratic," (E), is a new term,

and one the argument doesn't need.

2.  D

The Conclusion: None is stated, but as the question stem

a[e~ us, one is implied.

The Evidence: A plant opened in Country V during a

revolution ten years ago has a(ways generated substantial

profits.

The author draws an analogy between the two plants. Since

the one in Country Y has made money, so too, she implies,

will the one in Country X.

The author's point is that the plant can be successful

despite the war, (A), not that the plant will be successful

because of the war, The author presents the two countries

as similar, so she's not arguing that (g), one is more stable

than the other. The author is attacking her opponents'

argument, but not (C), their motives. She makes no

judgment, (E), as to which plant will be more successful.

Remember, comparing and contrasting things that are

considered equivalent in the stimulus is a common wrong

answer type for inference questions.

~,.  C

The Conclusion: There is none stated. One is implied,

though, and we're to locate it among the answer choices.

t        I Part Three: Practice Sets

162     Critical Reasoning Practice Set One

The Evidence: Since deregulation, low air fares have

correlated with increased miles flown. Recently, prices

have risen, but the number of passenger miles flown is stil

increasing.

The passage begins with a correlation and then breaks it in

the next sentence. What can you infer from this? Not very

much, you may have thought, and you'd be right. The key

to this question is not drawing an unwarranted sweeping

conclusion from the evidence. What you can safely

conclude is that the air fare doesn't, by itself, determine the

amount of air travel.

That air fares don't have any effect on air travel, so (A) is

too strong. We can't infer why passenger-miles flown have

increased, (B); it might be at the expense of other forms of

transportation, but it might not be. The author merely states

that the air fares have risen recently, but never mentions

a cause, (D), like "takeovers." So far, the recent rise in air

fares hasn't reduced passenger miles flown, but that doesn't

mean that a further increase won't, (E).

4.  A

The Conclusion: Consumers can be expected to switch

to Zydol.

The Evidence: Although one has to take two aspirins to get

the relief provided by one Zydol, nevertheless a bottle of

Zydol costs the same as a bottle of aspirin.

The implication is that Zydol is more cost-effective than

aspirin: You get a much better bargain with a bottle of Zydol

But if the aspirin bottle contains more than twice as many

tablets as the Zydol bottle, then it's the aspirin bottle that

gives you more pain reliever for your money.

The argument concerns which is the better bargain, not

which is the better pain reliever. Thus, it doesn't matter

(B) that the ingredient is the same. (C) points to some

consumers who won't want Zydol. Perhaps, but one, these

may be very few, and two, this doesn't attack the author's

reasoning, which is based on cost. Pain sufferers who don't

have a doctor's prescription are left with the same choice

between Zydol and aspirin, so (D) doesn't weaken Zydol's

case against aspirin. Even if you make an unwarranted

leap and assume (E), that there may be fewer of the large

Zydol pills per bottle, choice (A) clearly does a better job of

arguing along this line.

5.  B

The Undergraduates' Conclusion: If grades are assigned

by graduate assistants, regular full-time faculty will devote

less time to undergraduate work.

Their Evidence: There really isn't any; just the idea that

giving grading responsibilities to graduate students will

somehow move regular full-time faculty away from the

undergraduates' work.

The only thing to do is go through the choices to see which

question we can answer. Since the undergraduates are

against the proposal, it's safe to conclude that they want

the full-time faculty to stay interested in their work. We have

no information about the fairness of graduate assistants'

grades, (A). All we know about the discussion sections,

(C), is that graduate assistants now hold them; we aren't

told whether or not full-time faculty may hold them. (D)

asks about the validity of the faculty's grades. We know

that some people have proposed that graduate assistants

be given some grading responsibility, but that doesn't

imply (D), that they believe the faculty's grades have been

made less valid by the graduate students' "contact" with

undergraduates. And we're told absolutely nothing about

(E), the faculty's opinion of the proposal

6.  A

The Conclusion: Air traveI is becoming more dangerous.

The Evidence: In the last year, there have been seven

collisions and over 700 deaths, the highest number of

deaths ever.

We want something that suggests that the increase in

fatalities doesn't prove an increase in danger, if more and

more people are traveling by airplane, an increase in the

number of deaths doesn't prove that air travel is becoming

more dangerous. The question of how dangerous air travel

is can't be answered unless we know the proportion, not

the number, of passengers who get killed.

The author's contention isn't undermined by (B), the

reasons for the lack of safety. Pointing out more dangerous

methods of transportation, (C) doesn't deny that the danger

of air travel is increasing. Likewise, the fact that other forms

of transportation are also getting more dangerous, (D),

doesn't dent the author's claim. Fewer passengers per

place, (E), means more collisions, but we already know how

many collisions there were.
            7.  C

The Conclusion: Attending a single-sex high school

promotes growth.

The Evidence: During the first two years, the average

student in an all-boys school grew five inches and the

average girl in an all-girls school grew four inches.

The question stem tells you that something has been left

out of the argument. The conclusion compares single-sex

schools to toed schools, but the evidence only cites data

from single-sex schools. Until we know how much the

average student grows in co-educational schools, we have

no evidence that the growth in single-sex schools is greater

Even if we knew the answer to (A), we'd still have no

evidence from the coed schools. (B) is a rather silly question;

we're dealing with average growth, so we'd expect some

boys to grow more than the average and some less. The

reasons for the growth difference between boys and girls,

(D), isn't relevant to the growth differences between coed

and single-sex schools. Since the study is only interested in

how much students grow during the first two years of high

school, it's irrelevant how tall they are to start with, (E).

8.  E

The Conclusion: Exercise is good for the mind as well as

for the body.

The Evidence: Exercise increases the speed at which blood

circulates, allowing the brain to receive more oxygen and

thus concentrate better.

The main point is the conclusion, that exercise is good

for the mind as well as the body. More generally, exercise

serves more than one purpose.

(A) goes out on a limb; the author's point is the beneficial

effects of exercise, not the unlimited benefits of oxygen.

While exercise aids mental activity, it's not (B), a mental

activity as such. Whether exemise aids the brain more or

less than the rest of the body, (C), isn't discussed. This is a

classic wrong answer type; the choice that compares two

items the stimulus treats equally. While the author argues

that exercise is beneficial to the mind, "greatly improving"

concentration, (D), is an overstatement.

9.  E

Ernesto's Conclusion: VCR sales will decline,

Ernesto's Evidence: The saturation level for households

has been reached.

              Part Three: Practice Sets I

Critical Reasoning Practice Set OneI    163

Milton's Conclusion: VCR sales won't decline.

Milton's Evidence: More and more films are released on

videotape.

Milton simply ignores Ernesto's argument that the saturation

point has been reached for VCRs. He concentrates on

the new cassettes coming out, but the availability of

new cassettes doesn't mean people need to buy or will

buy more recorders. Milton ignores the relevant issue of

saturation raised by Ernesto, and simply assumes that sales

of VCRs will continue as before.

Milton's evidence doesn't disprove Ernesto's evidence,

(A). The videos appearing each year have nothing to

do with whether the saturation point in VCR ownership

has been reached. "Finding a gap," (B), in an argument

means demonstrating that the evidence doesn't lead to

the conclusion; Milton merely brings up some irrelevant

information that has nothing to do with Ernesto's reasoning.

Milton does cite an issue that Ernesto ignored, (C), but the

claim that this issue "outweighs" the issue of saturation is

unsupported. Milton does fail to speakto Ernesto's point,

(D), but not for the bizarre reason given here. Ernesto

has absolutely nothing to say about "other leisure-related

products" because there's no reason for Milton to raise this

ssue.

lO. A

The Conclusion: People must already have been using Type C weapons in this area by 2500 B.c.

The Evidence: Depictions of the weapons exist on the walls  of a cave that was sealed off in 2500 B.C. we need to show that the paintings either don't depict  weapons that were in use or that the paintings were made after 2500 aic, The archaeologists are assuming that there was only one entrance to the cave. ff there's another entrance to the cave that was only sealed much later, then people could have entered the cave and made the paintings long a~er the first entrance was sealed. if anything, the existence of the weapons in India, (B), might be considered to strengthen the argument, since it shows Type C weapons were in existence as early as 2500 B.C. Pushing the date of the eruption back, (C), also strengthens the argument by making it likely that the paintings we done even earlier than claimed. Without some evidence to back it up, the opinion of the experts, (D), isn't worth much. that identification was difficult, (E), is not the same as its being uncertain or controversial         

11. E

The Conclusion: Introducing a new cereal wiil only hurt the brands Coolidge already produces, so Coolidge shouldn't introduce another brand of cereal.The Evidence: The introduction of a new brand of cereal doesn't increase the total number of cereal buyers, but only encourages those who already buy cereal to switch brands. There's an assumption here: That the new brand will only attract those who currently buy other brands of Coolidge's cereal. If the new brand steals buyers from competitors' cereals, then it will help Coolidge by adding to its total sales. Even if the population does increase and totg cereal sales with it, (A), a new brand might still hurt the sales of Coolidge's established cereals. The new cereal may sell well, (B), but if that just means it's stealing lots of buyers from other Coolidge cereals, what good is it? The fact that Coolidge has only a few brands, (C), doesn't make it likely that a new cereal brand would steal buyers from competitors' brands rather than from other Coolidge brands. Neither does the fact that some brand switching is usual even when a new brand isn't introduced, (b).

12. D

The Conclusion: Not eating cheese protects children from

childhood diseases.

The Evidence: All the children at summer school who have

never had measles or chicken pox have also never eaten

the cheese served in the school lunches.

The author deduces a causal relationship: Not eating

cheese leads to protection from childhood diseases. I hope

you saw that this has numerous holes. Four of the choices

help to fill those holes; the fifth does not. That fifth choice is

(D). If research shows that abstaining from cheese products

is a ma)or cause of some childhood diseases, then the

health official's claim that children can protect themselves

from disease by not eating cheese is flat-oot wrong.

It's important that the author be able (A) to connect

measles and chicken pox (in the evidence) to other

childhood diseases. Also necessary is that the observations

be accurate, (B), and the students' behavior at home mirror

their behavior at school (C). Each of those speaks to the

legitimacy of the correlation. Most important, though, is that

there really be a causal connection, (E), rather than just a

correlation, between cheese eating and childhood illness.

13. D

The Consulting Firm's Conclusion: The employees

dissatisfaction is all in their heads.

Its Evidence: 75 percent of the employees are dissatisfied.

An investigation showed no uncommonly bad working

conditions.

We need the one choice that doesn't weaken the consulting

firm's conclusion; that's a good tip that the conclusion

isn't solidly based. The mere fact that the complaints vary

doesn't hurt the firm's conclusion that the complaints are

based on psychological problems. In fact, it might even

strengthen the firm's argument: Every employee has his

or her own mental hang-up, so every employee comes up

with his or her own irrational complaint.

If many employees had been working at a different

company's facilities, (A), then it could be these facilities, not

mental problems, that are responsible for the complaints. If

the situation in Company P is fairly normal, (B), there's no

need for the firm to hypothesize an unusual incidence of

psychological problems among the workforce. If the firm's

definition of uncommonly bad conditions differs from that

of the workers, (C), then the firm may well have overlooked

the real causes of employee dissatisfaction. And if a battery

of tests, (E), showed no significant psychological problems,

then the very cause cited by the firm is attacked.

14. B

The Director's Conclusion: We should build the dam at

the Bornos site.

Her Evidence: Although the same plant can produce the

same amount of electricity at either site, building at Abaco

will cost twice as much money.

Looking at the question stem first here really helps you

narrow your focus when reading the stimulus. The only

thing you're interested in is why the Abaco site will cost

more than twice as much as the Bornos site. The stimulus

doesn't give you any hint as to how the sites are different,

so you'll have to rely on the correct answer to provide a

complete explanation by itself. If the Abaco site is in the

middle of nowhere, and requires constructing new roads

and importing laborers, then, of course, it's going to be

more expensive to build there.

The benefits of the Bornos plant to farms, (A), don't explain

its lower cost of construction. Neither do the benefits to

nearby city residents, (C). Possible environmental damage
            caused by a plant built at Abaco, (D), is another reason for

building at Bomos, but it's a different reason. The problem

or issue of expansion, (El, like the environment, might merit

consideration, but it's not a factor in determining the cost of

building.

15. B

The Conclusion: These five people can serve as models for

the type of person who can be helped by hypnosis.

The Evidence: A study showed that these five previously

disturbed hypnosis subiedm had stable emotional

conditions.

The survey only found that at the time of the study the five

seemed to be doing okay. Remember, these people were

originally suffering from mood swings; maybe the study

just caught them on a good day. If that's the case-if since

  988 these people have been experiencing dramatic mood

swings and occasional periods of health-then hypnosis

hasn't really helped them and they're not good models.

The author presented the people as models of different

types of people who can be helped, not as a single model

of a single personality type, so they needn't be similar, (A)

]t doesn't matter that the other 45 people who underwent

hypnosis didn't get better (C); the argument is based

on and concerns only the 5 who were stable. (El fails to

weaken the argument for the same reason. The concern

of many psychologists, (D), is well outside the scope. We

need a statement that speaks about hypnosis and these

5 subjects.

16. A

/'he New Theory's Conclusion: Heat (from the burning

coal) causes columnar growth in iunipers near burning

underground coal veins.

The New Theory's Evidence: None really, except the

correlation of columnar growth with these areas with

underground fires.

The Old Theory's Conclusion: The abundance of carbon

monoxide causes columnar growth.

The Old Theory's Evidence: None really, except the

correlation of columnar growth with these areas with high

carbon monoxide.

When you scan the choices, you see that each presents

a case of the cause with or without the effect or the

effect with or without the cause. Since the two theories

              Part Three: Practice Sets

Critical Reasoning Practice Set One   

are in opposition, weakening the old theory is a way of

strengthening the new one. Bearing in mind from the lesson

the key issues in a causal argument, we recognize that a

case of columnar growth where the cause claimed by the

new theory (heat) is present, but the cause claimed by the

old theory (carbon monoxide) is absent, strengthens the

new theory at the expense of the old.

(B), where we get the new theo~/s alleged cause (intense

heat), without the alleged effect (columnar junipers), is of

no help at all. A columnar juniper Jn an atmosphere with

high carbon monoxide but no extra heat, (C), strengthens

the old theory. A case with both alleged causes without

the expected effect, (D), weakens both theories. Likewise,

columnar growth in the presence of both causes, (E), does

nothing to promote one theory over the other.

 


    

您需要登录后才可以回帖 登录 | 立即注册

Mark一下! 看一下! 顶楼主! 感谢分享! 快速回复:

手机版|ChaseDream|GMT+8, 2025-10-29 09:04
京公网安备11010202008513号 京ICP证101109号 京ICP备12012021号

ChaseDream 论坛

© 2003-2025 ChaseDream.com. All Rights Reserved.

返回顶部